Search found 6 matches

Return to advanced search

by yeh.briann
Mon Jun 08, 2015 2:58 pm
 
Forum: Section #2
Topic: Q20 - Ann will either take a leave
Replies: 21
Views: 7876
Jump to post

Re: Q20 - Ann will either take a leave

I understand andyevans000's explanation quite well, and I can see why D is the right answer because of his explanation. However, I am confused as to why, instead of taking the contrapositive of the conclusion, we could not have also taken the contrapositive of the premise that "Technocomp will ...
by yeh.briann
Wed Jun 10, 2015 8:33 pm
 
Forum: Section #3
Topic: Q9 - Lines can be parallel
Replies: 10
Views: 2642
Jump to post

Re: Q9 - Lines can be parallel

Is (c) wrong because it's a sufficient assumption?
by yeh.briann
Thu Jun 18, 2015 8:02 pm
 
Forum: Section #2
Topic: Q19 - In West Calverton, most pet stores
Replies: 16
Views: 7823
Jump to post

Re: Q19 - In West Calverton, most pet stores

Is (E) wrong because we can't conclude anything about independently-owned pet shops that don't sell tropical fish, or because we only know that independently-owned pet stores don't sell gerbils?

Thanks!
by yeh.briann
Fri Jun 19, 2015 10:34 am
 
Forum: Section #2
Topic: Q20 - Astronomer: Earlier estimates of the distance
Replies: 8
Views: 5507
Jump to post

Re: Q20 - Astronomer: Earlier estimates of the distance

Is (D) wrong for the additional reason that the actual distance of the stars should be fixed, so in reality it wouldn't differ between when the earlier estimates were taken and when the astronomer made his estimates?
by yeh.briann
Sat Jun 20, 2015 10:04 pm
 
Forum: Section #2
Topic: Q23 - Consumer advocate: TMD, a pesticide
Replies: 11
Views: 4327
Jump to post

Re: Q23 - Consumer advocate: TMD, a pesticide

I'd say (D) is wrong because it makes the argument solely about small children, whereas in the actual argument, they're just part of the reason pesticides are unacceptable. ("Others, including small children...") This argument is about pesticides being problematic for everyone , not just ...
by yeh.briann
Sun Jun 21, 2015 11:56 pm
 
Forum: Section #4
Topic: Q16 - Professor: Unfortunately, pharmaceutical companies
Replies: 4
Views: 3490
Jump to post

Re: Q16 - Professor: Unfortunately, pharmaceutical companies

I have a quick question. For answer choice (A), what is the logical opposite of "will"? Is it "will not" or "might not"? I understand why (A) is wrong for the reason given by patrick, but I'm trying to work through the negation of this AC as well.

Thanks!